Weaken Questions - - Question 49

The dean of computing must be respected by the academic staff and be competent to oversee the use of computers on cam...

JJennings January 27, 2014

confused

Please explain why E is the correct answer and not A.

Replies
Create a free account to read and take part in forum discussions.

Already have an account? log in

Naz January 29, 2014

Thank you for bringing this to our attention! The correct answer for this question should be (A), not (E). Please read below for a full explanation, we're sorry for any confusion the mislabeling caused!

The conclusion of the argument is: "Therefore, the dean of computing must be a professor from this university's computer science department."

C: DoC ==> PCSD
not PCSD ==> not DoC

Why? Because the the dean of computing must be respected by the academic staff and be competent to oversee the use of computers on campus."

P1: DoC ==> RA & COUC
not COUC or not RA ==> not DoC

And we know that deans who hold doctoral degrees are the only ones that academics respect.

P2: HDD ==> AR
not AR ==> not HDD

"and only someone who really knows about computers can competently oversee the use of computers on camps."

P3: COUC ==> RKC
not RKC ==> not COUC

We also know that "the board of trustees has decided that the dean of computing must be selected from among this university's staff."

P4: DoC ==> AUS
not AUS ==> not DoC

We want to weaken the argument.

Let's look at answer choice (A). "There are members of this university's staff who hold doctoral degrees and who are not professors but who really know about computers."

We want to weaken the conclusion that the author has come up with that the dean of computing must be a professor from this university's computer science department.

If there are other non-professor members of this university's staff who have the requirements necessary to become the new dean of computing, then that would weaken the conclusion that the dean of computing MUST BE a professor.

This is exactly what answer choice (A) is pointing out. We are saying that these non-professor members of the university's staff (so already we have checked off one requirement that the dean of computing must be selected from among this university's staff) have doctoral degrees (which we know means from P3 that they are respected by the academic staff--another requirement needed to be dean of computing) and they are really knowledgable about computers. Now, being really knowledgeable about computers does not necessarily man that they are competent to oversee the use of computers on campus, which is another requirement of being the dean of computing, but it also doesn't rule it out.

Meaning, this non-professor member of the university staff could potentially be a candidate for the dean of computing, i.e. it is no longer necessarily true that the dean of computing MUST BE a professor from this university's computer science department. We cannot necessarily rule out this group of non-professors mentioned in answer choice (A) using our principle rules from the passage, meaning that answer choice (A) has successfully weakened the argument.

Hope that clears things up! Please let us know if you have any other questions.

Derek July 15, 2014

So answer choice "A" would be correct if it added that a non-professor was respected by academics, had a doctorate, and knows about computers?

Naz July 17, 2014

The corrected explanation can be found in the thread above. Sorry for any confusion!

Hope that helps! Please let us know if you have any other questions.

NKWong May 19, 2015

For E, I am still a little confused how it weakens the argument, since it is just a "some" statement. Hypothetically, there can be a 1000 staff members in the computer science department, but only 1 or 2 of them are not respected by academics in other departments. Even if a subgroup is excluded from a larger group, I don't understand how it necessarily weakens the argument that a selection must be limited to that larger group. Basically, I initially eliminated E because I felt that that answer choice was too weak. How can avoid that mistake? Thanks!

Naz May 20, 2015

The corrected explanation can be found in the thread above. Sorry for any confusion!

Hope that helps! Please let us know if you have any other questions.

NKWong May 20, 2015

Thanks for the reply! Perhaps I am misinterpreting the conclusion incorrectly. As you pointed out from above, the conclusion of this prompt is "the dean of computing must be from this university's computer science department." When I read it the first time, I interpreted as a conditional statement, as in:
"The only person who can be the dean of computing is a professor from this university's computer science department."
OR
"If you are the dean of computing, then you are a professor from this university's computer science department."

In essence, I construed the word "must" as signifying the start of a necessary condition. Consequently, I thought either A or B was the answer, because they demonstrated that necessary condition did have to hold. I guess the LSAT intended them to be trap answers, but how can I recognize them as such? Thanks!

Naz May 20, 2015

The corrected explanation can be found in the thread above. Sorry for any confusion!

It is fine if you took the conclusion to be a Sufficient & Necessary statement: if dean of computing, then from the computer science department.

DoC ==> CSD
Not CSD ==> DoC

Answer choice (B) does nothing to the argument. The key is pinpointing whether or not these people are professors. Since we do not know anything about the university's philosphy department, i.e. whether they are all professors or not, we cannot infer anything from this answer choice. We are trying to debunk the conclusion that the dean of computing MUST BE a professor.

Hope that helps! Please let us know if you have any other questions.

mjhirsch August 25, 2015

Just to add to this conversation - the printed out questions/answers have the correct answer as (A), Are we sure E is correct?

Naz September 2, 2015

The corrected explanation can be found in the threads above. Sorry about the confusion!

Hope that clears things up! Please let us know if you have any other questions.

Flavio March 3, 2016

I picked answer choice A based on the idea of sufficient without necessary. Is this the right way of getting the correct answer for this question?

DanielDePasquale August 4, 2018

why is answer choice C wrong? I feel that it has the same effect as answer choice A does on the argument

MichelleRod August 4, 2018

Thanks for your question @DanielDelPasquale

These are the necessary criteria for the new dean of computing: holds a doctoral degree, really knows about computing, and is a member of the university staff.

Based on these criteria, the author concludes that "the dean of computing must be a professor from the university's computer science department.

Answer choice weakens the argument by showing us examples of potential candidates who meet all of the criteria specified in the stimulus without being professors.

The reason answer choice C doesn't work is because the example does not meet one of the criteria listed in the stimulus, namely that "the board of trustees has decided that the dean of computing must selected from among the university's staff."

Bhumi September 12, 2018

So A is the correct answer and not E?

Mehran September 24, 2018

@Bhumi yes the correct answer here is (A).

Cynthia-Lee October 2, 2018

Isn't "THE ONLY" is introduce sufficient condition. Weren't the sentence "And we know that deans who hold doctoral degrees are the only ones that academics respect." diagrammed as AR -> HDD ? I am confused about this diagramming, can someone please help to clarify this confusion, thanks.

alymathieu October 13, 2018

How are you supposed to figure that out in under two minutes that's ridiculous